0 Daumen
148 Aufrufe

Konvergiert folgendes Integral? \( \int\limits_{1}^{\infty} \) \( \frac{1}{(x-1)^2} \)

Einerseits denke ich nein, da wenn ich 1 einsetze das Integral nicht konvergieren kann.

Andererseits kann man behaupten, dass das Integral für alle x größer als 1 konvergiert.

Vielen Dank für die Aufklärung.

Avatar von

Wenn Du Dich mit dem Begriff "uneigentliches Integral" mal kurz beschäftigst, kannst Du Dir die Frage selbst beantworten.

1 Antwort

0 Daumen
 
Beste Antwort

 \( \int\limits_{2}^{\infty} \) \( \frac{1}{(x-1)^2} \) wäre nicht das Problem, aber

 \( \int\limits_{1}^{2} \) \( \frac{1}{(x-1)^2} \) divergiert.

Avatar von 54 k 🚀

Danke für deine Antwort,

folglich divergiert dann auch das Integral \( \int\limits_{1}^{\infty} \)\( \frac{1}{(x-1)2} \) oder?

Ein anderes Problem?

Stell deine Frage

Willkommen bei der Mathelounge! Stell deine Frage einfach und kostenlos

x
Made by a lovely community